LSAT and Law School Admissions Forum

Get expert LSAT preparation and law school admissions advice from PowerScore Test Preparation.

 smile22
  • Posts: 135
  • Joined: Jan 05, 2014
|
#14234
In this question, I incorrectly chose answer A. Could you please explain why this answer is not correct? The council member appears to be attacking the mayor's motives, so I thought he was making a "source" flaw.

When reading the stimulus again, the council member does attack the mayor's motives. He claims that the only reason the mayor is against imposing real estate development fees is because the mayor's family is heavily involved in real estate development. While this may be true, the mayor does legitimately reason that the real estate fees would significantly reduce the number of building starts, leading to revenue loss for the city. Based upon this, is answer A only half right? Answer B is a better answer choice because it addresses that the mayor does have personal interest for not passing the fees but also has factual data (the fees will result in revenue loss for the city) to back him up.
 Steve Stein
PowerScore Staff
  • PowerScore Staff
  • Posts: 1153
  • Joined: Apr 11, 2011
|
#14236
Hi,

That's a good question, and there's a reason that the test-makers decided to present that choice first--many test-takers found that to be an appealing wrong answer choice.

Take another look at the author's argument: Since the mayor has a personal interest in blocking development fees,he must be sacrificing the city's interests for his own.

What is the problem with the author's argument? Just because the mayor benefits from blocking such fees, that doesn't mean that the city can't benefit as well. That is what answer choice (B) provides.

Answer choice (A) states that the mayor's personal interest is irrelevant to any assessment of the mayor's action. That goes a bit too far. The author isn't saying that the mayor's personal interest is entirely irrelevant--just that the mayor's personal interest is not enough to conclude that the mayor is hurting the city. Subtle! Good question.

I hope that's helpful--please let me know--thanks!

Steve
 smile22
  • Posts: 135
  • Joined: Jan 05, 2014
|
#14260
Thank you for the explanation.
 adlindsey
  • Posts: 90
  • Joined: Oct 02, 2016
|
#42550
I had it between B and E and choose E. They both kind of sound the same to me.
 Claire Horan
PowerScore Staff
  • PowerScore Staff
  • Posts: 408
  • Joined: Apr 18, 2016
|
#42579
Hi adlindsey,

The conclusion of the argument is:
"It is clear that in doing so the mayor is sacrificing the city’s interests to personal interests."

This is identifiable from the words "it is clear."

Because the question stem asks us to identify why the argument is flawed, the correct answer choice must show why the author's conclusion doesn't follow.

The earlier posts in this thread explain why answer choice (B) identifies the flaw in the argument, so let's consider (E).

(E) reads: "the possibility remains open that the mayor’s need to avoid loss of family revenue is as desperate as the city’s need to increase municipal revenue"

Would it matter if the mayor's need was as desperate as the city's need? No! Regardless of how desperate his personal need is, if he acted to satisfy this need rather than the city's interests, the conclusion above, that he is "sacfricing the city's interests to personal interests" would still follow.

Get the most out of your LSAT Prep Plus subscription.

Analyze and track your performance with our Testing and Analytics Package.